Exam q's Flashcards

1
Q

The typical small investor would be unlikely to make a trade in which Market?
1,2,3 or 4

A

The fourth market is direct trades between investors. Because of the expense of trading in this market, only large institutions trade here.

How well did you know this?
1
Not at all
2
3
4
5
Perfectly
2
Q

Maturity and minimum denomination of the following:
Commercial Paper
Negotiable CD’s
Bankers Acceptances

A

Commercial Paper - maximum maturity is 270 days and the minimum denomination is usually $100,000, it can be lower but most institutional investors prefer to deal in minimums of $1M.

Negotiable CDs - usually short term matuirty used by money markets, banks will sell negotiable CDs with a minimum denomination of $100,000. However, trades in negotiable CDs have a minimum denomination of $1 million.

Bankers Acceptances - Usually trade in multiples of $100,000.

How well did you know this?
1
Not at all
2
3
4
5
Perfectly
3
Q

Series EE bonds carry a fixed-rate and are investments that are guaranteed to double in value over ….. years. The newer Series I bonds have both a fixed rate and a variable rate to keep up with …….?

A

Series EE savings bonds double in 20 years

Series I savings bonds have a fixed and variable rate to keep up with inflation

How well did you know this?
1
Not at all
2
3
4
5
Perfectly
4
Q

Mortgage Pass-Throughs

A

Pass through: In a mortgage pass-through, the issuer of the pass-through security collects the payments
from the borrowers, deducts a service charge, and then distributes the payments to the holders of the pass-through security. FNMA and FHLMC are the agencies that issue these securities.

How well did you know this?
1
Not at all
2
3
4
5
Perfectly
5
Q

Collateralized Mortgage Obligations

A

Collateralized mortgage obligations (CMOs) are similar to a pass-through, with one key difference. The issuer of a CMO sequences the securities. These sequences are called tranches. In a traditional CMO (known as a sequential pay CMO), interest will be paid on all of the tranches as promised, but the initial principal payments are directed to paying off only the first tranche. Once the first tranche has been paid in full, then the principal payments are directed to the owners of the second tranche. The holders of the first tranche
have a relatively short-term, low risk security. The holders of the last tranche own a security that has substantial credit risk and risk as to the timing of payments

How well did you know this?
1
Not at all
2
3
4
5
Perfectly
6
Q

With regard to collateralized mortgage obligations, which answer below is correct:
(A) The different tranches are of relatively equal safety.
(B) The maturity is fixed, just as with other bond-like instruments.
(C) The issuer is responsible for servicing these instruments.
(D) The principal received each period is relatively certain, the interest income is not.

A

The answer is (C). (A) is incorrect because although the first tranche is relatively safe, the later tranches can be quite risky. (B) is incorrect because the maturity is dependent upon when the borrowers pay off their mortgages. (D) is incorrect as interest and principal payments both depend on payments made by the mortgage borrowers, both are uncertain.

How well did you know this?
1
Not at all
2
3
4
5
Perfectly
7
Q
If the return on a security is normally distributed with an arithmetic mean of 6 percent and a standard deviation of 3 percent, then the probability of earning more than 12 percent is approximately?
(A) 5 percent
(B) 2½ percent
(C) 1 percent
(D) ½ percent
A

The answer is (B). The 12 percent is two standard deviations above the expected rate of return of 6 percent. The probability of a return more than two standard deviations from the expected return is 5 percent. However, this 5 percent is split equally between the upside and downside.

Approximately 68 percent of the time, the actual return will be within one standard deviation of the expected return

How well did you know this?
1
Not at all
2
3
4
5
Perfectly
8
Q

One of the most important things to remember about a normal distribution is that approximately 68 percent of the time, the actual return will be within …… standard deviation of the expected return and about 95 percent of the time the actual return will be within …… standard deviations of the expected return.

A

One of the most important things to remember about a normal distribution is that approximately 68 percent of the time, the actual return will be within one standard deviation of the expected return and about 95 percent of the time the actual return will be within two standard deviations of the expected return.

How well did you know this?
1
Not at all
2
3
4
5
Perfectly
9
Q
If the return on a security is normally distributed with an arithmetic mean of 6 percent and a standard deviation of 3 percent, then the probability of earning more than 12 percent is approximately
(A) 5 percent
(B) 2½ percent
(C) 1 percent
(D) ½ percent
A

The answer is (B). The 12 percent is two standard deviations above the expected rate of return of 6 percent. The probability of a return more than two standard deviations from the expected return is 5 percent. However, this 5 percent is split equally between the upside and downside.

How well did you know this?
1
Not at all
2
3
4
5
Perfectly
10
Q
You just bought a bond for 105 ($1,050). The bond will pay $70 in coupon payments annually. You plan to hold it to maturity in 10 years, and you believe you will be able to reinvest the coupons at a 5 percent rate of return. What would be your realized compound yield to maturity?
(A) 5 percent
(B) 5.5 percent
(C) 6 percent
(D) 6.5 percent
A

The answer is (C). The keystrokes for the realized compound yield to maturity are: SHIFT, C ALL; 10, N; 5, I/YR; 70, PMT; FV {Display: –880.45}; SHIFT, C ALL; 10, N; 1050, +/-, PV; 1880.45, FV; I/YR {Display: 6.00}

How well did you know this?
1
Not at all
2
3
4
5
Perfectly
11
Q

For a bond that pays $35 interest annually, trades at a price of 85 (i.e., $850), has a par value of $1,000, and a term to maturity of 8 years, the yield to maturity is

A

The answer is (B). The keystrokes for the yield to maturity are: SHIFT, C ALL; 8, N; 850, +/-, PV; 35, PMT; 1000, FV; I/YR {Display 5.91%}

How well did you know this?
1
Not at all
2
3
4
5
Perfectly
12
Q

Which of the following is correct with respect to what are known as alternative investments?
(A) a problem with trading collectibles is that there are no price indexes to get a sense of their change in value
(B) the depletion allowance for growing timber is an attractive tax incentive for investors
(C) the prices of collectibles are primarily a function of the demand curve
(D) gold jewelry is a good alternative investment as its value will move with the price of gold

A

The answer is (C). (A) is incorrect because both Barron’s and Forbes report the Sotheby index for prices of certain collectibles. (B) is incorrect because the depletion allowance applies to mineral rights, not timber. (D) is incorrect because although the price of jewelry will reflect, to some extent, the content of any precious metal such as gold, the price is mainly a function of the exquisiteness of the design and the currency of the style.

How well did you know this?
1
Not at all
2
3
4
5
Perfectly
13
Q
Suppose stocks X and Y have standard deviations of 5 percent and 10 percent and a covariance of 0. If 40 percent of a portfolio is invested in X and 60 percent in Y, what is the standard deviation of this two-security portfolio?
(A) 6.3 percent
(B) 7.5 percent
(C) 8.0 percent
(D) 40.0 percent
A

The answer is (A). The answer is calculated as follows:

Variance = (.402 x 52) + (.602 x 102) + (2 x .40 x .60 x 0) = 4 + 36 + 0 = 40
Standard deviation = Square root of 40 or 6.3 percent
Keystrokes: SHIFT, C ALL; .4, x, .4, x, 5, x, 5, =, M+; .6, x, .6, x, 10, x, 10, =, M+; 2, x, .4, x, .6, x, 0, =, M+; RM; SHIFT, yx, .5, = {Display: 6.3}

How well did you know this?
1
Not at all
2
3
4
5
Perfectly
14
Q

Which of the following statements concerning betas is correct?
(A) Stocks with negative betas have the least amount of risk.
(B) A stock with a beta equal to 1 has no risk.
(C) A stock with a beta of 0 is expected to provide a rate of return equal to the market portfolio.
(D) A stock with a beta greater than 1 is expected to be more volatile than the market portfolio.

A

The answer is (D). (A) is incorrect because a stock with a negative beta only means it moves opposite to the general market, not that it has the least risk. (B) is incorrect because a stock with a beta of 1 should move about the same as the overall market. (C) is incorrect because a stock with a beta of 0 would provide a rate of return about the same as the risk-free rate.

How well did you know this?
1
Not at all
2
3
4
5
Perfectly
15
Q

What is Modern Portfolio Theory and why is it important to understand?

A

Modern portfolio theory (MPT) is a theory on how risk-averse investors can construct portfolios to optimize or maximize expected return based on a given level of market risk, emphasizing that risk is an inherent part of higher reward. According to the theory, it’s possible to construct an “efficient frontier” of optimal portfolios offering the maximum possible expected return for a given level of risk.

It is important to understand because it is the basis for how most professionals think about the market.

How well did you know this?
1
Not at all
2
3
4
5
Perfectly
16
Q

Which of the following is a feature of the S&P 500 Index?
(A) it is the most commonly quoted index
(B) the index includes common stocks, preferred stocks, and warrants, but not rights
(C) the adjustment for dividends paid in shares of stock (i.e., stock dividends) is always complicated
(D) it is a value-weighted index

A

The answer is (D). (A) is incorrect because the most commonly quoted index is the Dow Jones Industrial Average. (B) is incorrect because the index includes only common stocks. (C) is incorrect because, as a value-weighted index, dividends paid in shares of stock do not alter the market capitalization’s of any of the companies.

How well did you know this?
1
Not at all
2
3
4
5
Perfectly
17
Q

Market capitalization is

A

Market capitalization is equal to the share price multiplied by the number of shares outstanding.

How well did you know this?
1
Not at all
2
3
4
5
Perfectly
18
Q

The volatility of a bond’s price when interest rates change is greater
(A) when the term to maturity is shorter.
(B) when the coupon rate is higher.
(C) when its duration statistic is smaller.
(D) when the yield to maturity is lower.

A

The answer is (D). (A), (B) and (C) are associated with reduced volatility in a bond’s price when interest rates change.

How well did you know this?
1
Not at all
2
3
4
5
Perfectly
19
Q

Which of the following is a violation of the wash sale rule?
(A) You sell 27 shares of an S&P 500 index fund at a loss. Then, 15 days later, you buy 42 shares of a Russell 3000 index fund.
(B) You have brokerage accounts at two firms. At the first firm you buy 100 shares of PQR stock. Then, 22 days later, you sell 100 shares of PQR stock, bought the prior year, for a loss in your account at the other firm.
(C) Your spouse buys 200 shares of UK stock. Then, 31 days later, you sell 100 shares of the same stock, bought 43 days earlier, at a slight loss.
(D) You sell 100 shares of UK stock at a slight gain. Then, 4 days later, your spouse buys 200 shares of the same stock.

A

The answer is (B). (A) is incorrect because the two indices are sufficiently different. (C) is incorrect because the purchase is more than 30 days prior to the sale. (D) is incorrect because the stock is sold at a gain.

How well did you know this?
1
Not at all
2
3
4
5
Perfectly
20
Q

Growth investing in stocks typically means investing in which of the following?
(A) above-average-P/E stocks
(B) below-average-P/E stocks
(C) stocks with recent large jumps in their dividend payments
(D) stocks with large recent price increases

A

The answer is (A). (B) is incorrect because it is the definition of value investing. (C) and (D) are incorrect because they are meaningless as descriptions of investment styles.

How well did you know this?
1
Not at all
2
3
4
5
Perfectly
21
Q

Which statement best describes the relationship between the overall economy and the stock market?

(A) There is no discernible relationship between the two.
(B) The stock market typically leads the overall economy.
(C) The stock market typically moves with the overall economy.
(D) The stock market typically lags behind the overall economy.

A

The answer is (B). The stock market is one of the leading indicators identified by the National Bureau of Economic Research (NBER).

How well did you know this?
1
Not at all
2
3
4
5
Perfectly
22
Q

Which of the following is correct with regard to measuring a company’s profitability?
(A) a firm with a weak ROA could still have an average or better ROE if its equity multiplier is large enough
(B) ROE indicates the profits earned relative to the amount of total assets owned by the firm
(C) if two firms in the same industry have the same ROE, then it really doesn’t matter what their ROA ratios are
(D) the equity multiplier is one of the more effective profitability ratios

A

The answer is (A). (B) is incorrect because ROE is profits relative to equity, not total assets. (C) is incorrect because as ROE = ROA x Equity Multiplier, which means a firm with a weak ROA that uses extensive debt financing (and this has a large equity multiplier), is much weaker than a firm with a strong ROA and little use of debt financing. (D) is incorrect because the equity multiplier is a financial leverage ratio, not a profitability ratio.

How well did you know this?
1
Not at all
2
3
4
5
Perfectly
23
Q

Which of the following statements regarding duration is correct?
(A) The higher the coupon rate, the larger the duration statistic.
(B) The duration of a zero-coupon bond equals its term to maturity.
(C) Usually, the longer the term to maturity, the shorter the duration statistic.
(D) The higher the yield to maturity, the larger the duration statistic.

A

The answer is (B). (A) and (D) are incorrect because duration is inversely related to the coupon rate and yield to maturity. (C) is incorrect because duration is normally directly related to term to maturity.

Duration: It is analogous to the beta statistic for a stock, wherein the higher the beta, the more volatile
the price of the asset for a given change in the market portfolio.

How well did you know this?
1
Not at all
2
3
4
5
Perfectly
24
Q
If a standard call on MM stock with a strike price of $20 is purchased for $3 per share, the buyer exercises at the close of trading on the expiration date, and the stock closes at $30 per share, then what is the buyer's profit before taxes and commissions
(A) $700
(B) $1,000
(C) $2,000
(D) $2,300
A

The answer is (A). A standard call option is for 100 shares. If the stock closes at $30 and the strike price is $20, the buyer of the call has a gain of $10 per share, or $1,000 on the stock itself. However, the buyer paid $300 for the option, so the buyer’s net gain is $700 ($1,000 – $300).

How well did you know this?
1
Not at all
2
3
4
5
Perfectly
25
Q

Which of the following statements concerning warrants is correct?

(A) The exercise of warrants involves the issuance of new shares by the company.
(B) They typically have a longer term to expiration than rights, but a shorter term to expiration than LEAPS®.
(C) Companies sell them with the hope that they will expire worthless.
(D) When they are issued in conjunction with a bond offering, the bond’s coupon rate is normally higher.

A

The answer is (A). (B) is incorrect because, not only do warrants have longer terms to expiration than rights, they also typically have longer terms to expiration than LEAPS®. (C) is incorrect because companies issue warrants with the hope that they will be exercised in the future, thus allowing the company to sell stock at a price greater than the current price. (D) is incorrect because attaching the warrants to a bond allows a lower coupon rate to be provided due to the extra benefit of the warrants.

How well did you know this?
1
Not at all
2
3
4
5
Perfectly
26
Q

Long-term Equity Anticipation SecuritiesSM (LEAPS®)

A

Puts and calls are normally written for relatively short periods (9 months is typically the longest available); however, a 1990 innovation in the options market called Long-term Equity Anticipation Securities SM (LEAPS®)—were introduced on the CBOE and the AMEX and they are now traded on all the options exchanges. There are no differences in the terminology or strategy used with LEAPS® as with regular options, other than that LEAPS® can have a substantially longer time to expiration. There are two types of LEAPS®: Equity LEAPS® and Index LEAPS®. Equity LEAPS® have expiration dates as long as 3 years from the date of initial listing. Index LEAPS® may have expiration dates as long as 5 years from the date of the initial listing.

How well did you know this?
1
Not at all
2
3
4
5
Perfectly
27
Q

For a bond that pays $55 interest annually, trades at a price of 90 (i.e., $900), has a par value of $1,000, is callable in 5 years at a price of $1,055, and matures in 10 years, the yield to maturity is

A

6.92%.
The keystrokes for the yield to maturity are: SHIFT, C ALL; 10, N; 900, +/-, PV; 55, PMT; 1000, FV; I/YR {Display: 6.92%}

How well did you know this?
1
Not at all
2
3
4
5
Perfectly
28
Q

A cereal manufacturer needs 50,000 bushels of corn next month. The spot (cash market) price for corn is $5 per bushel, and the futures contract with delivery next month has a price of $5.50 per bushel. The manufacturer goes long contracts for 50,000 bushels. Which of the following statements is correct?
(A) The current basis is $.50.
(B) If the basis does not change, the manufacturer will effectively buy the corn for $275,000.
(C) If the basis does not change and the spot price of corn falls, the manufacturer will be glad it hedged with the futures contract.
(D) If the basis decreases (i.e., becomes more negative), the manufacturer will actually make a profit on its hedge.

A

The answer is (D). (A) is incorrect because the basis is the cash price minus futures price, which, in this case, is –$.50. (B) is incorrect because the manufacturer will buy the corn at the current spot price, or $250,000 ($5 x 50,000 bushels) if the basis does not change. (C) is incorrect because the manufacturer will realize it could have bought the corn for a cheaper price if it had not hedged.

How well did you know this?
1
Not at all
2
3
4
5
Perfectly
29
Q

Which of the following statements about financial futures contracts is correct?
(A) A person expecting interest rates to rise and who wants to speculate on that expectation might go long an interest rate futures contract.
(B) The futures contract on 10-year T-notes requires delivery of a T-note with a maturity of 10 years.
(C) Stock market index futures contracts only exist for the S&P 500 index.
(D) The S&P 500 futures contract is worth 250 times the S&P 500 index.

A

The answer is (D). (A) is incorrect because if interest rates rise, the price of the underlying bond would fall, and the price of the interest rate futures contract would also most likely fall. (B) is incorrect because any T-note with a maturity greater than 6 1/2 years may be delivered. (C) is incorrect because there are at least six different indexes covered by futures contracts.

How well did you know this?
1
Not at all
2
3
4
5
Perfectly
30
Q

Which of the following accurately describes tactical asset allocation decisions?
(A) These decisions are made for the purpose of establishing the risk exposure of the portfolio.
(B) These decisions are made only occasionally.
(C) These decisions may include more asset categories than are found in strategic asset allocation decisions.
(D) In an organization, these decisions are made at the highest level.

A

The answer is (C). (A), (B), and (D) are incorrect because they describe strategic asset allocation decisions.

The main difference between strategic and tactical asset allocation is that tactical asset allocation is used to make changes to an investor’s asset allocation based primarily on short-term capital market expectations. In contrast, strategic asset allocation focuses only on long-term capital market expectations. Tactical asset allocation is used to try to beat the market. Strategic asset allocation is used to assure the riskiness of the portfolio matches the investor’s risk tolerance.

How well did you know this?
1
Not at all
2
3
4
5
Perfectly
31
Q

Which of the following statements regarding the psychological issues of investing is correct?

(A) People are more likely to invest when given more choices.
(B) There is a natural tendency for people to see patterns where none exist.
(C) People who check the value of their investments more frequently tend to do better.
(D) Experienced investors usually hold more diversified portfolios.

A

The answer is (B). (A) is incorrect because more choices typically confuse people and they are less likely to invest. (C) is incorrect because people who check their values more frequently tend to trade unnecessarily. (D) is incorrect because experienced investors tend to be overconfident about their investment selections, and concentrate their holdings.

How well did you know this?
1
Not at all
2
3
4
5
Perfectly
32
Q
  1. Which of the following is (are) correct about corporate bankruptcy?
    I. Chapter VII bankruptcy involves the company obtaining court protection while it reorganizes.
    II. Any of several parties can initiate a bankruptcy proceeding.
A

II only
I is incorrect because Chapter VII is for liquidation, Chapter XI is for reorganization.

VII = 7 (liquidating)
XI = 11 (reorganizing)
How well did you know this?
1
Not at all
2
3
4
5
Perfectly
33
Q

What is a Red Herring?

A

To initiate a public offering, the issuing firm and its investment banker compose a registration statement. This is submitted to the Securities and Exchange Commission (SEC) for review. During the review period, the investment banker may distribute this registration statement in an effort to drum up initial interest in the offering. Because the front page of the statement contains a paragraph in red ink indicating that the company is not attempting to sell its shares before the SEC approves the registration, the statement is known as a red herring. Red herrings are sometimes revised several times before the issue is ready for sale to the public.

How well did you know this?
1
Not at all
2
3
4
5
Perfectly
34
Q

Which of the following statements about investment banking is (are) correct?
I. When a red herring is issued, it means the investment banker is signaling the offering is highly speculative.
II. In a firm commitment, the underwriter buys the securities for resale to the public.

A

II only.

I is incorrect because a red herring is a preliminary prospectus used to drum up interest in an offering.

How well did you know this?
1
Not at all
2
3
4
5
Perfectly
35
Q

Which of the following statements concerning specialists is (are) correct?
I. Specialists make a market in listed stocks.
II. Specialists maintain a book of limit orders to be executed.

A

Both I and II

How well did you know this?
1
Not at all
2
3
4
5
Perfectly
36
Q

A year ago, Sally bought some stock for $40. During the year, she received $2 in dividends, which she took as cash. Now, at the end of the year, her stock is worth $48. Which of the following statements is (are) correct?

I. Her holding period return (HPR) is 5 percent because she has not yet sold the stock and the ending value is therefore speculative.
II. Her holding period return relative (HPRR) is 1.20 percent because return relatives do not include dividends or interest.

A

Neither I nor II

I is incorrect because the HPR is 25 percent ([8 + 2] / 40); it is immaterial whether or not the holding is actually sold. II is incorrect because the HPRR is then 1.25 (1 plus the HPR).

How well did you know this?
1
Not at all
2
3
4
5
Perfectly
37
Q

Which of the following statements regarding short sales is (are) correct?
I. The lender of the stock for a short sale loses any dividends that would otherwise accrue.
II. These transactions can be made only after a downtick.

A

Neither I nor II

I is incorrect because the borrower of the stock must make good any dividends the lender would have received. II is incorrect because, except under special conditions, short sales may take place anytime.

How well did you know this?
1
Not at all
2
3
4
5
Perfectly
38
Q

Which of the following statements about the capital market line (CML) is (are) correct?
I. The CML defines the expected return for any security or portfolio.
II. The CML is defined by combinations of a risk-free asset and the market portfolio.

A

II only

I is incorrect because the CML is relevant only for fully diversified portfolios.

Capital market line (CML) is a graph that reflects the expected return of a portfolio consisting of all possible proportions between the market portfolio and a risk-free asset.

How well did you know this?
1
Not at all
2
3
4
5
Perfectly
39
Q

You have the following holdings with the associated beta values:

 Stock                Market Value             Beta

ABC Co. $30,000 2.0

XYZ Ltd. $30,000 1.4

WJW Inc.                $60,000                    .9

Which of the following is (are) correct?

 I.    The portfolio's beta is 1.3.

II.    The most volatile stock in the portfolio is the ABC Co.
A

Both I and II are correct.

To compute the portfolio’s beta, first determine the percentage allocation to each asset, then multiply those percentage allocation by the associated betas, and then sum the products. The calculation is:

Name Weight Beta Product
ABC Co. .25 2.0 .5
XYZ Ltd. .25 1.4 .35
WJW Inc. .5 .9 .45
Sum 1.3

How well did you know this?
1
Not at all
2
3
4
5
Perfectly
40
Q

Over the last 3 months, a portfolio has rates of return of 8 percent, 4 percent, and –2 percent. Which of the following is (are) correct about its performance?

I. the rate of return for the entire quarter is 10.07 percent

II. the geometric mean monthly rate of return is 3.25 percent

A

Both I and II

To compute the rate of return for the entire quarter, first convert the per period returns to return relatives, then multiply them together, and then subtract one. The computation is: 1.08 x 1.04 x .98 – 1 = .1007 or 10.07%. To compute the geometric mean return, first convert the per period returns to return relatives, then multiply them together, take the cube root (because there are three time periods), and then subtract one. The computation is: (1.08 x 1.04 x .98)1/3 – 1 = .0325 or 3.25%

How well did you know this?
1
Not at all
2
3
4
5
Perfectly
41
Q

The Normandy Fund had a return of 14 percent, a standard deviation of 12 percent, and a beta of 1.2. During the same period, the market had a return of 12 percent and a standard deviation of 8 percent. The risk-free rate was 6 percent. Which of the following statements concerning Normandy’s performance is (are) correct?
I. Normandy’s Sharpe ratio was 0.67.
II. Normandy’s “Jensen’s alpha” was 0.008, or .8 percent.

A

Both I and II

Sharpe ratio . 14 . 06
. 12
0.67 p f
p

Jensen’s alpha = Rp − [Rf + βp(RM − Rf)]
= .14 − [.06 + 1.2(.12 − .06)]
= .14 − .132
= 0.008 or .8%

How well did you know this?
1
Not at all
2
3
4
5
Perfectly
42
Q

Which of the following statements is (are) correct with respect to common stock?
I. Class A common stock would always be preferred to Class B common stock.
II. When voting by proxies, investors can always vote for or against the recommendation of the Board of Directors.

A

I only

I is incorrect because there are no legal definitions as to what is meant by a Class A or Class B share. Depending on the terms of the shares for a particular company, some investors may prefer Class A and others may prefer Class B.

How well did you know this?
1
Not at all
2
3
4
5
Perfectly
43
Q

Which of the following is (are) correct with respect to time-weighted rates of return?

I. A time-weighted rate of return is always less than a dollar-weighted rate of return over the same time period.
II. If a portfolio earns 20% in the first year and loses 20% in the second year, the time-weighted rate of return is –2.02%.

A

II only

I is incorrect because, depending on when the cash inflows and outflows occurred, a time-weighted rate of return could be higher or lower than a dollar-weighted rate of return.

How well did you know this?
1
Not at all
2
3
4
5
Perfectly
44
Q

According to the textbook, when selecting a mutual fund, advisors should, along with other criteria, look for funds:
I. with the best historical performance in terms of total returns.
II. with the lowest fees.

A

II only

I is incorrect because there is no significant correlation between past and future performance, other than for the worst performing funds, past performance should not be a significant factor in the selection of a fund.

How well did you know this?
1
Not at all
2
3
4
5
Perfectly
45
Q

Which of the following statements is (are) correct with respect to the mechanics of paying a dividend?
I. The ex-dividend date precedes the record date.
II. Firms typically increase their dividends in the same quarter of each year, whichever quarter that is.

A

Both I and II are correct.

Decoration date –> Ex-dividend date –> Record date –> payment date

Ex dividend date is set by the exchange and need to buy shares before this date if you want to receive a dividend.

How well did you know this?
1
Not at all
2
3
4
5
Perfectly
46
Q

Which of the following statements about monetary policy is (are) correct?
I. Open market operations are used by the Federal Reserve Board on a daily basis to influence the level of bank reserves.
II. The Federal Reserve Board adjusts reserve requirements almost as frequently as it adjusts its discount rate.

A

I only

II is incorrect because the Federal Reserve Board rarely changes the reserve requirements; such changes are potentially quite disruptive to the banking industry.

How well did you know this?
1
Not at all
2
3
4
5
Perfectly
47
Q

You just received a margin call for $5,000. You can meet the margin call in which of the following ways, assuming a 25% maintenance margin rate?

I. Sell $5,000 worth of stock.

II. Deposit $5,000 in cash.

A

II only

If you sell $5,000 worth of stock, the loan balance goes down by $5,000, but the market value of your account also goes down by $5,000. For example, suppose you had a $40,000 debit balance and your account balance fell to $46,666. In this case, your margin call would be $5,000.

Margin call = loan – [MV x (1 – MMR)] = $40,000 – [$46,666 x (1 – .25)] = $40,000 – 35,000 = $5,000

If you sell $5,000 in stock and pay down the loan, your loan balance is then $35,000 and the market value of your account is $41,666. You would still be subject to the following margin call: $35,000 – [$41,666 x (1 – .25)] = $35,000 – $31,249.50 = $3,750.50. In other words, you need to sell a lot more stock to meet a margin call than just what the amount of the call is.

How well did you know this?
1
Not at all
2
3
4
5
Perfectly
48
Q

Which of the following statements concerning a bond’s yield to maturity is (are) correct?
I. It is the discount rate that causes the present value of the coupon payments and the par value to equal the price of the bond.
II. It is normally found by dividing a bond’s coupon rate by its current market price.

A

I only

II is incorrect because the coupon rate divided by the current price is the definition of the current yield.

How well did you know this?
1
Not at all
2
3
4
5
Perfectly
49
Q

Which of the following statements regarding bond swaps is (are) correct?

I. An example of a tax swap is if an investor sells a bond at a loss and immediately buys a similar but not substantially identical bond.
II. Selling a bond and replacing it with a similar bond that has a higher current yield is a pure-yield pick-up swap.

A

Both I and II are correct.

How well did you know this?
1
Not at all
2
3
4
5
Perfectly
50
Q

Which of the following statements concerning cost basis is (are) correct?
I. Commissions are deductible expenses, not part of the cost basis.
II. Any accrued interest paid on the purchase of a bond is offset against other interest income.

A

II only

Commissions are added to the cost basis of any asset purchased.

How well did you know this?
1
Not at all
2
3
4
5
Perfectly
51
Q

An investment has a 40 percent probability of a 20 percent rate of return, a 35 percent probability of a 0 percent rate of return, and a 25 percent probability of a -10 percent rate of return. Which of the following is (are) correct?
I. The expected rate of return is 5.5 percent.
II. The returns approximate a symmetrical, bell-shaped distribution.

A

I only

II is incorrect because in this example the highest probability is associated with the highest rate of return and the lowest probability with the lowest rate of return. If one draws a histogram, one will see that the distribution is negatively skewed.

How well did you know this?
1
Not at all
2
3
4
5
Perfectly
52
Q

Even though funds, on average, underperform market indices, they can still be good investments for which of the following reasons?

I. Any mutual fund investment is guaranteed to provide adequate diversification.
II. For smaller accounts, they may provide the only effective vehicle for diversification.

A

II only

I is incorrect because some mutual funds concentrate on stocks from just one sector, industry, or country.

How well did you know this?
1
Not at all
2
3
4
5
Perfectly
53
Q

Which of the following statements regarding the Black-Scholes option pricing model is (are) correct?
I. As the risk-free interest rate increases, a call option becomes more valuable.
II. The longer the time to expiration, the more valuable the call option is.

A

Both I and II

How well did you know this?
1
Not at all
2
3
4
5
Perfectly
54
Q

Which of the following statements concerning convertible bonds is (are) correct?
I. They provide an inexpensive way to buy stock.
II. They provide a lower yield than do straight bonds of comparable credit risk and maturity because of the attractiveness of the conversion feature.

A

II Only

I is incorrect because convertible bonds normally trade at a premium to conversion value. Thus, buying a convertible bond and then converting provides an investor with fewer shares of stock than if the investor had just bought the stock directly to start with.

How well did you know this?
1
Not at all
2
3
4
5
Perfectly
55
Q

Which of the following statements concerning futures contracts is (are) correct?
I. Whether one is taking a long or short position, a futures contract requires a deposit analogous to earnest money; no margin loans are involved.
II. Futures contracts can be used for hedging, although hedging limits the potential of windfall gains from favorable price movements.

A

Both I and II

How well did you know this?
1
Not at all
2
3
4
5
Perfectly
56
Q

With regard to the concept of immunizing a bond portfolio, which of the following statements is (are) correct?
I. One of the main goals of immunization is to minimize the variability of the prices of the bonds in the portfolio.
II. Perfect immunization is possible only if there is a one-time-only change in interest rates.

A

II only
I is incorrect because an immunized portfolio may still have price variability. The goal of immunization is only to lock in the yield to maturity for one’s desired holding period.

Price risk: The risk of an existing bond’s price changing when market interest rates change. If rates increase, the bond’s price decreases, and if rates decrease, the bond’s price increases.Reinvestment rate risk: The risk associated with reinvesting coupon payments as market interest rates change. If rates increase, the coupons
are reinvested at higher rates than previously expected, and if rates decrease, the coupons are reinvested at lower rates than previously expected. A portfolio is immunized when the expected gains from one of these changes exactly off set the expected losses
from the other change.

How well did you know this?
1
Not at all
2
3
4
5
Perfectly
57
Q

Which of the following statements regarding the taxation of options is (are) correct?
I. The writer of an option must declare the premium as a capital gain at the time of writing.
II. If the owner of a call exercises the option, the option premium is subtracted from the strike price and commissions paid in determining the investor’s basis on the acquired stock.

A

Neither I nor II

I is incorrect because the tax treatment of a premium is not determined until the option position is closed out. Hence, at the time of writing, there is no tax recognition of the premium received. II is incorrect because when the owner of a call exercises the option, he or she is buying the stock. Hence, the cost basis of the stock is the premium PLUS the exercise price plus the commissions paid.

How well did you know this?
1
Not at all
2
3
4
5
Perfectly
58
Q

Which of the following statements regarding investment strategies for tax-advantaged accounts is (are) correct?
I. Net unrealized appreciation refers to the increase in value on stocks that have not yet been sold in a nonqualified account.
II. For optimal tax efficiency, one should place securities with current income in qualified accounts and securities with good prospects for capital gains in nonqualified accounts.

A

II only

I is incorrect because net unrealized appreciation is the difference between the market value of the stock and the cost basis of that stock when shares of one’s employers’s stock are withdrawn from a tax-qualified account.

59
Q

Which of the following statements concerning the constant dividend growth rate model is (are) correct?
I. The constant growth rate model implies that a stock’s price will increase at the same rate at which dividends are increasing.
II. The model works best when the dividend growth rate exceeds the required rate of return.

A

I only

II is incorrect because if the growth rate exceeds the required rate of return, which is the same as the discount rate, then the denominator is negative, and this would produce a negative stock price.

Dividend Growth Model (Gordon’s model) Gordan’s model states that dividends are expected to grow at a constant rate, g, forever.

60
Q

All of the following statements concerning closed-end investment companies are correct EXCEPT

(A) The share price is based on supply and demand, and rarely equals the net asset value (NAV).
(B) The portfolios are usually actively managed.
(C) An investor will pay a normal commission, rather than a load charge, when buying these shares.
(D) The number of shares outstanding will change as investors buy new shares or redeem existing shares.

A

The answer is (D). The number of shares of a closed-end fund is normally fixed at the time the fund is established.

The major distinction among investment companies is whether they are open-ended or closed-ended. Open-ended investment companies are known popularly as mutual funds. The distinction between a mutual fund and a closed-end company is that the only way an investor can buy shares in a mutual fund is from the fund itself through the creation of new shares. The only way an investor can sell shares of a mutual fund is back to the fund, which is referred to as redeeming shares. Shares in a closed-end investment company can be bought only from another investor; they also can be sold only to another investor.

61
Q

All of the following statements about money market instruments are correct EXCEPT
(A) Because they are two-named paper, bankers’ acceptances have one of the lowest yields of the money market instruments.
(B) There is no trading in bankers’ acceptances during the last 14 days.
(C) The maximum maturity on commercial paper is 270 days.
(D) Both competitive and noncompetitive bids are accepted on newly issued Treasury bills.

A

The answer is (B). Negotiable CD’s cease trading when there are 14 days to maturity, not negotiable bankers’ acceptances.

You can think of a negotiable instrument (such as a promissory note) as one that can be transferred from its holder to a third party. Non-negotiable instruments are governed by applicable common law. Most significantly, a non-negotiable instrument typically cannot be transferred from its holder to a third party.

62
Q

For the following Treasury bond quotation:

   Rate      Mat.       Bid        Ask       Ask Yld.

   5.00      Apr 20   101:15   101:19       4.88

All of the following statements concerning the above are correct EXCEPT

(A) The coupon rate is 5 percent.
(B) The bond matures on April 20th.
(C) The bond can be bought for 101 19/32 percent of par.
(D) The bid-ask spread is 4/32 of 1 percent of par.

A

The answer is (B). The bond matures in April of 2020.

63
Q

All of the following are accurate descriptions of the Securities Investor Protection Act of 1970 EXCEPT
(A) A joint account has coverage up to $1,000,000.
(B) It does not cover commodity contracts.
(C) Not all broker/dealers registered with the SEC have SIPC coverage.
(D) It does not protect the investor from the consequences of being hacked.

A

(A) is wrong because the maximum coverage is $500,000 per account, even if it is a joint account.

64
Q

All of the following statements regarding orders are correct EXCEPT
(A) A limit order to buy specifies the maximum price the investor will pay.
(B) A market order to sell will be executed only on an uptick.
(C) A limit order may not be executed.
(D) Market and limit orders are the most commonly used types of orders.

A

The answer is (B). A market order is executed immediately, regardless of the direction of the tick. It used to be that short sales could only be executed on upticks, but this rule was abolished in 2007.

65
Q

Over the past 2 years, an investment has provided annual rates of return of +10 percent and -10 percent. All of the following statements are correct EXCEPT

(A) The arithmetic mean return is 0 percent.
(B) The geometric mean return is –.5 percent.
(C) The variance of return is 100 percent-squared.
(D) The standard deviation of return is 14.14 percent.

A

The arithmetic mean is (10% – 10%)/2 = 0.

The geometric mean is [(1 + .10)(1 – .10)] .5 – 1 = –.005, or –.5%. Keystrokes: Shift, C All; 1.1, x, .9, =; Shift, yx, .5, =; –, 1, =

The variance is [(10% – 0)2 + (–10% – 0)2]/(2 – 1) = 200 percent-squared.

The keystrokes are:

SHIFT, C ALL; 10, Σ+; 10, +/– Σ+; SHIFT, sxsy; SHIFT, x2

66
Q

All of the following statements regarding the selection of the optimal portfolio to hold are correct EXCEPT
(A) An investor would be happy with any portfolio that lies below the capital market line.
(B) In theory, any risky portfolio (other than the market portfolio) should be inferior to one or more combinations of the risk-free asset and the market portfolio.
(C) A portfolio tangent to a single indifference curve would be the optimal one to hold.
(D) A portfolio whose standard deviation is smaller than that for the market portfolio would have a lower expected return than the expected return on the market portfolio.

A

The answer is (A). (A) is incorrect because the investor will always want a portfolio ON the capital market line, not underneath it.

Capital market line (CML) is a graph that reflects the expected return of a portfolio consisting of all possible proportions between the market portfolio and a risk-free asset.

The market portfolio is the portfolio of all assets, with the weight of each based on its market value.

67
Q

All of the following statements regarding Monte Carlo simulations are correct EXCEPT
(A) The user only needs to specify expected rate of return for the portfolio.
(B) They can be used for accumulation portfolios.
(C) They can be used for decumulation portfolios.
(D) The output is frequently a distribution of final portfolio values.

A

The answer is (A). The standard deviation of the portfolio’s rate of return is just as important as the expected rate of return.

68
Q

With respect to risk-adjusted returns, all of the following statements are correct EXCEPT
(A) One should only compare risk-adjusted returns, not actual returns.
(B) The information ratio is quite sensitive to the values of the betas of the portfolios being evaluated.
(C) The Sharpe ratio uses standard deviation as the measure of risk.
(D) The Treynor ratio and Jensen’s alpha use beta as the measure of risk.

A

The answer is (B). The information ratio does not depend on the value of a portfolio’s beta.

The information ratio looks at how a portfolio manager performed relative to an index or benchmark. It is sometimes referred to as the appraisal ratio. Let’s say that a portfolio manager’s objective is to outperform the S&P 500 index. To do so, the manager needs to
adjust his portfolio to be slightly different from the portfolio defined by the index.

69
Q

What is the information ratio

A

The information ratio looks at how a portfolio manager performed relative to an index or benchmark. It is sometimes referred to as the appraisal ratio. Let’s say that a portfolio manager’s objective is to outperform the S&P 500 index. To do so, the manager needs to
adjust his portfolio to be slightly different from the portfolio defined by the index.

70
Q

All of the following are correct statements about tests for weak-form efficiency EXCEPT
(A) The evidence on serial correlation coefficients is mixed.
(B) Filter rules work for only very small filters.
(C) Volume changes are moderately accurate predictors of future price changes.
(D) Tests of runs–numbers of consecutive days of price changes in the same direction–show no unusual patterns.

A

The answer is (C). There are no known tests that show any relationship between volume changes and subsequent price changes.

71
Q
  • Current stock prices fully reflect all publicly available information. Future returns are unrelated to any analysis based on public information. (Fundamental analysis does not result in superior returns.)
  • Current stock prices fully reflect all market trading data, including past stock prices, trading volume, and short sales. Future returns are unrelated to past return patterns. (Charting and other types of technical analysis do not produce superior returns.)
  • Current stock prices fully reflect all information. Future returns are unrelated to any analysis based on public or nonpublic data. (Insider trading does not result in superior returns.)

1) Weak
2) Semi-Strong
3) Strong

A

1) Weak form: Current stock prices fully reflect all market trading data, including past stock prices, trading volume, and short sales. Future returns are unrelated to past return patterns. (Charting and other types of technical analysis do not produce superior returns.)
2) Semi-Strong: Current stock prices fully reflect all publicly available information. Future returns are unrelated to any analysis based on public information. (Fundamental analysis does not result in superior returns.)
3) Strong: Current stock prices fully reflect all information. Future returns are unrelated to any analysis based on public or nonpublic data. (Insider trading does not result in superior returns.)

72
Q

All of the following statements regarding deducting losses on IRA accounts are correct EXCEPT
(A) There is a 10 percent penalty if the account is closed out before the investor turns 59 1/2.
(B) Any losses taken are reduced by up to 2 percent of the investor’s adjusted gross income.
(C) When taking a loss, all IRA accounts of the same type must be closed out.
(D) This tactic can apply to any of the different types of IRA accounts.

A

The answer is (A). No penalty would apply as there is no gain that is being recognized.

73
Q
The major premises of the chartist's approach to analyzing a stock include all of the following EXCEPT
(A) Stock prices follow trends.
(B) Company analysis is crucial.
(C) Volume follows the price trend.
(D) Resistance and support levels exist.
A

The answer is (B). Chartists generally do not rely on company analysis when making their investment decisions.

74
Q

All of the following statements are correct with respect to the capital market line EXCEPT
(A) An investor can never hold a portfolio riskier than the market portfolio.
(B) Some combination of the risk-free asset and the market portfolio will always be preferred to a portfolio of N risky assets (excluding the market portfolio itself).
(C) The expected rate of return is 7 percent for a portfolio whose standard deviation is 6 percent, when the standard deviation of the market portfolio is 16 percent, the risk-free rate is 4 percent, and the expected return on the market portfolio is 12 percent.
(D) Risk-averse investors will tend to put more into the risk-free asset and less into the market portfolio.

A

The answer is (A). (A) is incorrect because to create a portfolio riskier than the market portfolio one only has to buy the market portfolio on margin.

75
Q

You are considering an investment that you believe has a 25 percent probability of a –15 percent rate of return, a 40 percent probability of a 5 percent rate of return, and a 35 percent probability of a 25 percent rate of return. What is the expected rate of return on this investment?

A

Expected return = .25 x (–15) + .40 x 5 + .35 x 25 = 7.00
Keystrokes: SHIFT, C ALL; .25, x, 15, +/–, =, M+; .40, x, 5, =, M+; .35, x, 25, =, M+; RM
{Display: 7.0000}
—————————————————————————-
E (R) = (P1 × R1) + (P2 × R 2) + . . . + (Pn × R n)
where E(R) = expected rate of return
Pi = the probability of each rate of return occurring
Ri = each potential rate of return
n = the number of possible rates of return

76
Q

All of the following are correct statements about preferred stock EXCEPT
(A) Preferred stock always has a promised dividend payment.
(B) Most preferred stock is owned by corporations.
(C) In the event of bankruptcy, preferred stockholders are grouped with general creditors.
(D) A company may issue multiple classes of preferred stock.

A

The answer is (C). Preferred stockholders are ranked after general creditors, but before common stockholders.

77
Q

To slow down the rate of economic expansion, Congress might engage in all of the following fiscal policy actions EXCEPT
(A) raise income taxes
(B) lower federal spending
(C) increase marginal tax rates
(D) enlarge the income brackets for each marginal tax rate

A

The answer is (D). (D) is incorrect because by increasing the income brackets, Congress would be effectively lowering everyone’s tax liabilities.

78
Q

All of the following are correct statements regarding tax-loss harvesting and tax-efficient investing EXCEPT
(A) Selling a security in a nonqualified account at a loss to deduct the loss from one’s ordinary income is tax-loss harvesting.
(B) Tax-efficient investing can lead to concentrated portfolio holdings.
(C) Tax-loss harvesting means taking any losses as soon as they occur.
(D) It might be tax-efficient to take capital gains to offset capital losses that might be lost due to the expected death of the investor.

A

The answer is (C). Tax-loss harvesting means taking losses that can be used to offset capital gains or current income, not taking any and all losses that suddenly appear. Tax-loss harvesting is usually undertaken once or twice a year and usually toward the end of the calendar year.

79
Q
All of the following would be expected to increase the value of a bond EXCEPT
(A) restrictions on dividend payments
(B) a conversion feature
(C) a call feature
(D) a sinking fund provision
A

The answer is (C). If a bond is called, it is normally replaced with another that has a lower coupon rate. This tends to decrease the value of a callable bond.

80
Q

All of the following are correct statements about portfolio rebalancing EXCEPT

(A) Optimal rebalancing involves a trade-off between transaction costs and the psychological cost of being on an inferior indifference curve.
(B) The simplest rebalancing strategy is calendar rebalancing.
(C) The alternative to calendar rebalancing is to rebalance any time an asset category moves outside its target range.
(D) Portfolio rebalancing could involve changes in the strategic or tactical asset allocations.

A

The answer is (D). Rebalancing means bringing the asset allocations back into alignment with the strategic and tactical assets allocations, not changing them.

81
Q

All of the following are correct statements about selling a client on a plan EXCEPT
(A) Clients tend to like the idea that a portfolio will have enough liquidity to avoid selling when prices have declined.
(B) Clients typically like to see strategies that show the cash they will need from the portfolio over the next few years is safely invested.
(C) Buffer zone plans are less likely to fail than plans that restrict investments only to bonds.
(D) It is important to emphasize the technical aspects of the plan, including expected return and standard deviation of return.

A

The answer is (D). Clients tend to be turned off by an emphasis on the technical aspects of an investment plan, and respond better to a conceptualization of how a plan might work.

82
Q

All of the following statements about UITs are correct EXCEPT
(A) All equity UITs must have a termination date, at which time the portfolio must be liquidated.
(B) UITs are typically set up and marketed by a brokerage firm that receives an underwriting fee from the proceeds of the sale and manages the trust until the trust is liquidated.
(C) Although there is frequent trading by the portfolio manager in an equity UIT, the only trading in debt UITs involves pure-yield pick-up swaps.
(D) Investors technically buy units rather than shares.

A

The answer is (C). There is minimal to no trading in either debt or equity UITs, but, to the extent a debt UIT makes a trade, it is usually because of a default on a bond in the portfolio.

83
Q

When a common stock’s price is less than the exercise price of a put on that stock, all of the following statements are true EXCEPT
(A) The put option is trading in the money.
(B) The put option has an intrinsic value to it.
(C) There is a reasonably good chance the put option will be exercised, especially if the expiration date is near.
(D) The premium for the call option with the same exercise price and expiration date is almost certainly greater than the premium for the put option.

A

The answer is (D). As the market price of the stock falls, the premium of the put option becomes larger and the premium of the call option falls.

84
Q

All of the following statements about options are correct EXCEPT
(A) If the expected price on the expiration date of the underlying stock is less than the exercise price of a call option, then a call option is worthless.
(B) The intrinsic value for call options is the greater of the difference between the market price and the exercise price or zero.
(C) Options have a speculative value to them, which is based on the potential variability of the price of the underlying stock between now and the expiration date.
(D) Options are a form of speculative investing, somewhat akin to buying on margin.

A

The answer is (A). The speculative value of an option depends not on the expected price on the expiration date of the stock, but rather on the probability that the price of the stock might end up above the exercise price and the magnitude of that spread.

85
Q

All of the following statements concerning commodities futures markets are correct EXCEPT

(A) Margins of 50 percent or more are typical.
(B) Short and long positions are always numerically equal.
(C) Positions are marked to market on a daily basis.
(D) Actual payment would take place at delivery, if there is a delivery.

A

The answer is (A). Margins in commodities futures markets are generally between 5 percent and 15 percent.

86
Q

All of the following are correct with regard to goals and implementation of monetary and fiscal policy EXCEPT

(A) The two have the same major goals.
(B) Price stability is a major goal.
(C) Full employment is a major goal.
(D) Both are controlled by professional economists.

A

The answer is (D). (D) is incorrect as monetary policy is controlled by the Federal Reserve Board, and fiscal policy is controlled by Congress. Although the Fed may be dominated by professional economists, few people in Congress have had any education in economics.

87
Q

All of the following are correct about the treatment of capital gains and losses EXCEPT

(A) Long-term capital gains are taxed at a lower rate than short-term capital gains for most taxpayers.
(B) In a given year, a taxpayer can deduct up to $3,000 in long-term capital losses and up to $3,000 in short-term capital losses.
(C) An investment that is sold exactly one year after the purchase date is treated as having a short-term holding period.
(D) The sale price for a capital asset is the proceeds less the commission paid for the sale.

A

The answer is (B). (B) is incorrect because the maximum deduction against ordinary income in any one year is $3,000 for most taxpayers (the exception is for those whose status is married filing separately), whether it is short-term losses, long-term losses, or some combination of the two.

88
Q

All of the following are correct with regard to ETFs and Index Funds EXCEPT
(A) Index funds can be shorted, ETFs cannot.
(B) ETFs can be traded anytime the market is open.
(C) Index funds can always be purchased at their NAV, but ETFs may trade at prices higher or lower than their NAV.
(D) Not all ETFs are tied to some sort of index.

A

The answer is (A). (A) is incorrect as ETFs trade on organized exchanges in the secondary markets and therefore can normally be sold short. Index funds cannot be shorted.

89
Q

All of the following regarding portfolio performance are correct EXCEPT

(A) A mutual fund with a beta coefficient of 1.10 is less volatile than the market.
(B) Even though the risk-free rate is 3%, a portfolio with a Jensen’s alpha of 2% has “beaten” the market.
(C) The higher the value of the information ratio, the better the portfolio manager’s performance.
(D) One portfolio could have a higher information ratio than a second portfolio, but a lower Jensen’s alpha.

A

The answer is (A). (A) is incorrect because a mutual fund, or any asset, with a beta of 1.10 is expected to be slightly more volatile than the market.

90
Q

All of the following are correct with respect to investing in mutual funds EXCEPT

(A) At least once a year there is a capital gains distribution if the fund has accumulated net capital gains.
(B) A fund with an accumulated net capital loss provides an opportunity for shielding future capital gains within the fund.
(C) Any dividends distributed by a mutual fund are treated as qualified.
(D) Tax-friendly mutual funds will be characterized typically by low portfolio turnover ratios.

A

The answer is (C). Dividend distributions retain their original tax treatment. So qualified and nonqualified dividends are distributed as such, and tax-exempt interest income is still tax exempt to the mutual fund investor.

91
Q

Which one of the following is an implicit cost of trading?
(A) price impact

(B) commissions

(C) cost of investment advice

(D) margin account interest

A

The answer is (A).
(B) is incorrect because commissions are an explicit cost of trading, and (C) and (D) are incorrect because the cost of investment advice and interest in a margin account are not considered costs of trading.

92
Q

Which of the following statements about stock index options is correct?
(A) Taking long positions subject the buyer to a maximum possible loss that is greater than the option premium.
(B) They allow investors to trade on general stock market movements.
(C) They are good only for speculation, not hedging.
(D) Because no actual deliverable exists, they can only be closed out with the sale of the option (for those who are long) or repurchased (for those who are short).

A

The answer is (B). (A) is incorrect because the maximum loss to one who is long equals the option premium. (C) is incorrect because stock index options are just as good for hedging as speculation. (D) is incorrect because they can be settled in cash at expiration.

93
Q
A convertible bond that matures in 10 years has a par value of $1,000, trades at $900, and has a conversion price of $20. What is the market price of the company's common stock?
(A) $20
(B) most likely between $18 and $20
(C) $18
(D) most likely less than $18
A

The answer is (D).
A conversion price of $20 means the conversion ratio is 50 ($1,000/$20). If the conversion premium were zero, the market price of the stock would be $18 ($900/50). Since the conversion premium is most likely greater than zero because the bond has 10 years to maturity, the price of the common stock is most likely less than $18.

94
Q

Which of the following statements regarding the futures market is correct?
(A) Margin calls can result from the marking to market.
(B) Commodities futures margins range from 20 percent to 50 percent.
(C) Investors can close out a position with a different brokerage firm, but they would have to pay the round-trip fee a second time.
(D) Daily price limits protect investors from any significant losses.

A

The answer is (A).
(B) is incorrect because futures margins are usually in the 5 to 15 percent range.
(C) is incorrect because investors must use the same firm to close out a position.
(D) is incorrect because daily price limits only spread a loss out over time; they do not prevent it from happening.

95
Q
A cereal producer goes long 10 contracts for corn (5,000 bushels per contract). The price of the futures contract is $5 per bushel and the cash price at the time of the trade is $4 per bushel. When it is time to buy the corn in the cash market, the cash price is $4.50, and the futures price is also $4.50. The effective cost of the purchase, after adjusting for the gain or loss on the futures contract would be:
(A) $25,000
(B) $200,000
(C) $225,000
(D) $250,000
A

The answer is (D).
The cash purchase price is $4.50 per bushel, or $225,000. The producer went long at $5 per bushel, and sold the contract at $4.50 per bushel. So it lost $.50 per bushel, or $25,000. The effective purchase price is the $225,000 plus the $25,000 loss, or $250,000.

96
Q
If you have a debit balance of $25,000 and the maintenance margin rate is 25 percent, what is the minimum market value of assets you could have before you would be exposed to a margin call?
(A) $25,000
(B) $31,250
(C) $33,333
(D) $40,000
A

The answer is (C).

The minimum market value is obtained by dividing the loan balance by (1 – maintenance margin rate). Thus, $25,000/(1 – .25) = $25,000/.75 = $33,333.

97
Q

A tax aspect of investing in mutual funds is which of the following?

(A) The capital gain distribution is always taxed in the year it is received.
(B) There are multiple methods of tracking the cost bases of mutual fund shares.
(C) The long-term or short-term treatment of the capital gain distribution is based on how long the investor has held the shares.
(D) All dividend distributions are treated as qualified dividends.

A

The answer is (B). (A) is incorrect because capital gains distributions received in January are allocated to the prior year for taxation purposes. (C) is incorrect because the holding period is based on how long the fund has held the shares, not the investor. (D) is incorrect because the qualified, non-qualified distinction is passed through to the investor.

98
Q

A DCA plan works best in cyclical markets.

A

True

99
Q

In dividend reinvestment plans, investors avoid the taxes on the dividend income.

A

False

100
Q

DCA plans are usually superior to plunking if one has a choice.

A

False. If a choice is available, plunking will usually produce superior returns.

101
Q

Interest rate risk consists of reinvestment rate risk and price risk.

A

True

102
Q

Correlation Coefficient

A

The Correlation Coefficient. The correlation coefficient is measured on a scale from -1 to 1. A correlation coefficient of 1 indicates a perfect positive correlation between the prices of two stocks, meaning the stocks always move the same direction by the same amount.

103
Q

why we pay so much attention to modern portfolio theory

A

It provides the theory supporting the Sharpe ratio.

It is the most commonly accepted theory about stock market returns that currently exists.

104
Q

ETFs have all the trading characteristics of any other listed stock. Hence, they can be shorted, bought on margin, and so forth.

A

True

105
Q

It takes longer to make changes in monetary policy than in fiscal policy.

A

False. monetary policy can be placed into service quicker than fiscal policy.

106
Q

Monetary policy is more difficult to track than fiscal policy.

A

False because the figures on money supply are released weekly.

107
Q

Which of the following statements regarding business cycles is (are) correct?
I. The leading indicators series has the most components of the three series of indicators published by the Conference Board.
II. The leading indicators series occasionally gives false signals.

A

Both I and II are correct.

108
Q

Bond Immunization

A

Bond immunization is an investment strategy used to minimize the interest rate risk of bond investments by adjusting the portfolio duration to match the investor’s investment time horizon. … In other words, the bond is “immune” to fluctuating interest rates.

109
Q

Which of the following statements is (are) correct with respect to bond immunization?
I. Immunization occurs when the gains or losses in price due to a change in interest rates exactly offsets the losses or gains in reinvestment opportunities due to the same change.
II. A bond portfolio is considered immunized when its duration equals the investor’s intended holding period.

A

Both I & II

110
Q

Federal funds are deposits by commercial banks at Federal Reserve Banks

A

True

111
Q

Which of the following statements concerning open-end investment companies is (are) correct?
I. Their shares usually trade at a discount to the net asset value (NAV).
II. They issue new shares to buyers, and redeem shares from sellers.

A

II only

I is incorrect because the shares always trade exactly at NAV.

112
Q

Which of the following statements concerning the stock of closed-end investment companies is (are) correct?
I. The price of its shares tends to sell at a premium over NAV.
II. An investor has the choice of selling or redeeming his or her shares.

A

Neither

I is incorrect because the majority of closed-end funds trade at discounts, not premiums. II is incorrect because investors can only sell their shares–they are not given a redemption option.

113
Q

The price of DT stock is $70, and the market expectation is that the price in 6 months will be $75. A 6-month call option with an exercise price of $80 trades at a price of $1.50 per share. Which of the following statements is (are) correct with regard to this call option?
I. The intrinsic value is zero.
II. It has a speculative or time value because of the possibility that the price might move above $80 at some point during the next 6 months.

A

Both are correct

Puts and calls always have an intrinsic value. For an in the money option, intrinsic value is the difference between the market price of the underlying stock and the exercise price. For at the money or out of the money options, the intrinsic value is zero.

114
Q

It is usually more efficient to rebalance portfolios when they move outside of the targeted range, than on a fixed calendar basis.

A

True

115
Q

Which of the following statements concerning the federal tax treatment of capital gains and losses is (are) correct?
I. Generally, up to $3,000 of net capital losses may be deducted from ordinary income with any remaining losses carried over to future tax years.
II. There is no taxation of capital gains on stocks and bonds until an investor’s position in a security is closed out, assuming it is closed out at a profit.

A

Both I and II

116
Q

With regard to the taxation of bonds, which of the following statements is (are) correct?
I. It is possible to have a capital gain or capital loss on a zero-coupon bond.
II. The accrued interest paid on the purchase of a bond is an itemized deduction.

A

I only

II is incorrect because the accrued interest paid is deducted against the interest income that is later paid to the owner of the bond.

117
Q

Investors make better trading decisions on Friday than on Mondays.

A

False

There is no evidence that the day of the week has any impact on decision-making skills.

118
Q

People sometimes make investment errors by overgeneralizing from a small sample.

A

True

119
Q

Securitization

A

Securitization involves taking assets that heretofore were not easily traded in a secondary market and structuring a marketable security or group of securities from them. The goal of the process is to convert assets with poor marketability into assets with much greater
market acceptance.

The institution doing the securitization usually sells the new securities for appreciably more than what it cost to buy the underlying assets. In addition, the issuer usually charges a service fee on the new securities.

Mortgages are the primary asset that is securitized.

120
Q

What is a bullet portfolio?

A

A bullet portfolio is one in which the entire portfolio is placed in one maturity

121
Q

A portfolio with 20 percent invested in bonds with maturities of 1 through 5 years each is a laddered portfolio.

A

True

122
Q

A portfolio that has 50 percent invested in bonds with a 1-year maturity and the other 50 percent invested in bonds with a 10-year maturity is a barbell portfolio.

A

True

123
Q

A portfolio with 100 percent of the money invested in bonds whose maturities are all 5 years is a bullet portfolio.

A

True

124
Q

Any bond portfolio can provide immunization if the portfolio’s duration matches the investor’s intended holding period.

A

True

125
Q

Covariance

A

The covariance, like the mean and standard deviation, is a statistic that is almost always estimated from ex post (historical) values of the relevant variables. It measures the comovement or covariability of two variables.

126
Q

The higher the covariance, the greater the benefits from diversification

A

False.
The lower the covariance, the greater the benefits from diversification. In fact, negative covariances provide excellent diversification opportunities.

127
Q

When faced with choosing among portfolios, a risk-averse investor will always make the following choices EXCEPT

(A) He or she will always choose the less-risky portfolio if the expected returns are equal.
(B) He or she will always choose the portfolio with the greater expected return if two portfolios have equal risk.
(C) If one portfolio has both greater expected return and less risk than a second portfolio, he or she will always choose the first portfolio.
(D) He or she will always choose a risk-free asset over a risky portfolio.

A

The answer is (D).
Risk aversion does not mean total risk avoidance. A risk-averse investor will choose a risky asset over a risk-free one if the incremental expected return is sufficient to compensate him or her for the extra risk.

128
Q

Equity REITs invest in properties that produce income or have growth potential

A

True

129
Q

Mortgage REITs hold mostly construction loans and mortgages on commercial property

A

True

130
Q

RELPs lost their popularity as a result of changes in the tax code that eliminated many of the benefits of this type of investment.

A

True

131
Q

The principal and interest payments of REMICs are not passed through to investors on a pro rata basis because REMICs utilize tranches.

A

True

132
Q

The constant growth model:

  • The constant growth model assumes a constant growth rate forever, not a constant change in the growth rate.
  • It provides investment analysts with a tool for estimating the intrinsic value of common stock
  • It requires the use of the next period’s dividend in the numerator, rather than the most recent dividend
  • It requires that the discount rate exceed the growth rate.
A

True

133
Q

All of the following statements regarding financial analysis are correct EXCEPT
(A) The current ratio is used to assess the capital structure of the firm’s balance sheet.
(B) The short-form version of Du Pont analysis breaks return on equity (ROE) into the two components of return on assets (ROA) and equity multiplier.
(C) A high total asset turnover ratio may indicate that a company should invest in more assets.
(D) A low average collection period may indicate that the company needs to be more generous with regard to its credit policies.

A

The incorrect answer is A
The current ratio is used to assess liquidity. Capital structure is assessed by such ratios as the debt ratio or debt-equity ratio.

134
Q

All of the following are correct with regard to the purchase of mutual fund shares EXCEPT

(A) A letter of intent can lead to lower load fees.
(B) The right of accumulation applies to the other accounts the investor owns with that sponsor.
(C) Higher breakpoints are associated with lower load fees.
(D) By law, load fees cannot exceed 10 percent, although maximum loads today are about 3 percent.

A

The answer is (D). By law, load fees cannot exceed 8.5 percent, although maximum loads today on equity funds are about 5.75 percent and on bond funds around 4.75 percent.

135
Q

With respect to securities and interest rates related to the money market, all of the following are correct EXCEPT
(A) Repurchase agreements are the sale of securities with an option to repurchase within 3 days.
(B) LIBOR is sometimes used by banks as an index for its variable rate loans.
(C) BANs, RANs, and TANs are forms of short-term municipal bonds.
(D) The discount rate is the rate the Fed charges for loans to commercial banks.

A

The answer is (A).

Repurchase agreements are normally overnight loans, although they can have longer maturities.

136
Q

All the following statements about corporate bonds are correct EXCEPT
(A) An original-issue discount (OID) bond has a coupon rate that was less than the market rate at the time of issuance.
(B) Most corporate bonds trade in the OTC market.
(C) A debenture has no specific assets pledged as collateral.
(D) Collateral trust bonds are used to finance rolling stock such as airplanes.

A

The answer is (D).
Collateral trust bonds have financial assets pledged as collateral. Equipment trust certificates are used to finance rolling stock.

137
Q
Factors that affect a bond's yield to maturity include all of the following EXCEPT
(A) par value
(B) sinking fund provisions
(C) death puts
(D) call protection
A

The answer is (A).
The par value affects price, but not the yield.

Sinking fund provisions: Sinking funds reduce the probability of default, thereby tending to enhance a bond’s value.

138
Q

Factors Affecting Bond Yields

A

Factors Affecting Bond Yields
• General credit conditions: Credit conditions affect all yields to one degree or another.
• Default risk: Riskier issues require higher promised yields.
• Term structure: Yields vary with maturity, reflecting expectations of future interest rate changes.
• Marketability: Actively traded issues tend to be worth more than otherwise equivalent issues that are less actively traded.
• Seasoning: Newly issued bonds may sell at a slight discount to otherwise equivalent established issues.
• Call protection: Protection from an early call tends to enhance a bond’s value.
• Sinking fund provisions: Sinking funds reduce the probability of default, thereby tending to enhance a bond’s value.
• Me-first rules: Bonds protected from the diluting effect of additional borrowings are generally worth more than otherwise equivalent unprotected issues.

139
Q

All of the following statements are correct about unit investment trusts (UITs) EXCEPT
(A) UITs can be composed of bond or equity portfolios.
(B) There is a date at which the UIT is supposed to go out of existence.
(C) The secondary market for ownership of these trusts is quite active.
(D) Management fees of these trusts tend to be low due to the lack of a need for active portfolio management.

A

The answer is (C).
Issuers of UIT units may redeem these instruments, but there is no active secondary market in the trading of these instruments.

140
Q

NAV calculation

A

It is computed as the market value of the portfolio minus any liabilities of the fund, divided by the number of shares outstanding.

141
Q

For investment companies who hold securities that trade only sporadically, the NAV is an approximation.

A

True

142
Q

The NAV can be an overstatement of value if the fund has substantial unrecognized capital gains

A

True

143
Q

The NAV may move dramatically, especially if the market has made a major move.

A

True